Last visit was: 23 Apr 2024, 12:55 It is currently 23 Apr 2024, 12:55

Close
GMAT Club Daily Prep
Thank you for using the timer - this advanced tool can estimate your performance and suggest more practice questions. We have subscribed you to Daily Prep Questions via email.

Customized
for You

we will pick new questions that match your level based on your Timer History

Track
Your Progress

every week, we’ll send you an estimated GMAT score based on your performance

Practice
Pays

we will pick new questions that match your level based on your Timer History
Not interested in getting valuable practice questions and articles delivered to your email? No problem, unsubscribe here.
Close
Request Expert Reply
Confirm Cancel
SORT BY:
Date
Tags:
Show Tags
Hide Tags
User avatar
Intern
Intern
Joined: 11 Jan 2007
Posts: 29
Own Kudos [?]: 157 [66]
Given Kudos: 4
Location: United States
Concentration: Marketing, Healthcare
GMAT 1: 600 Q49 V25
GMAT 2: 650 Q49 V32
GPA: 3.5
WE:Pharmaceuticals (Consulting)
Send PM
Most Helpful Reply
User avatar
Manager
Manager
Joined: 04 May 2010
Posts: 61
Own Kudos [?]: 333 [17]
Given Kudos: 7
 Q51  V44
GPA: 3.8
WE 1: 2 yrs - Oilfield Service
Send PM
Tutor
Joined: 16 Oct 2010
Posts: 14816
Own Kudos [?]: 64880 [12]
Given Kudos: 426
Location: Pune, India
Send PM
General Discussion
avatar
Intern
Intern
Joined: 12 Jul 2010
Status:fighting hard..
Posts: 16
Own Kudos [?]: 24 [0]
Given Kudos: 24
Concentration: general
Schools:ISB, Hass, Ross, NYU Stern
 Q50  V31 GMAT 2: 700  Q50  V35
Send PM
Re: Contrary to the statements of labor leaders, the central economic prob [#permalink]
Thanks a lot Abhay Prasanna. Superb explanation..
Intern
Intern
Joined: 25 Jul 2012
Posts: 19
Own Kudos [?]: 27 [0]
Given Kudos: 144
Location: India
Concentration: Finance, Marketing
GMAT 1: 660 Q44 V37
Send PM
Re: Contrary to the statements of labor leaders, the central economic prob [#permalink]
I am not able to figure why A is the correct answer. I marked B
Manager
Manager
Joined: 23 Aug 2021
Posts: 216
Own Kudos [?]: 145 [0]
Given Kudos: 75
Send PM
Re: Contrary to the statements of labor leaders, the central economic prob [#permalink]
Which of the following statements by a labor leader focuses on the logical weakness in the argument above?

So the statement above is stating that The wealth distribution to the working class is small because of stagnation of productivity. But the fact that the distribution of the pie to the working class is small is not justified.
(A) Although the economic pie is no longer growing, the portion of the pie allocated to American workers remains unjustly small. This is the Answer
Manager
Manager
Joined: 10 Jul 2021
Posts: 233
Own Kudos [?]: 48 [0]
Given Kudos: 29
Send PM
Re: Contrary to the statements of labor leaders, the central economic prob [#permalink]
VeritasKarishma wrote:
shaneforu wrote:
Contrary to the statements of labor leaders, the central economic problem facing America today is not the distribution of wealth. It is productivity. With the productivity of U.S. industry stagnant, or even declining slightly, the economic pie is no longer growing. Labor leaders, of course, point to what they consider an unfair distribution of the slices of pie to justify their demands for further increases in wages and benefits. And in the past, when the pie was still growing, management could afford to acquiesce. No longer. Until productivity resumes its growth, there can be no justification for further increases in the compensation of workers.

Which of the following statements by a labor leader focuses on the logical weakness in the argument above?


(A) Although the economic pie is no longer growing, the portion of the pie allocated to American workers remains unjustly small.

(B) If management fails to accommodate the demands of workers, labor leaders will be forced to call strikes that will cripple the operation of industry.

(C) Although productivity is stagnant, the U.S. population is growing, so that the absolute size of the economic pie continues to grow as well.

(D) As a labor leader, I can be concerned only with the needs of working people, not with the problems faced by management.

(E) The stagnation of U.S. industry has been caused largely by factors—such as foreign competition—beyond the control of American workers.


Argument of management:

- Productivity stagnant so economic pie not growing
- Was pie was growing, management could increase wages.
- Now management can't afford

Conclusion: Until productivity resumes its growth, there can be no justification for further increases in the compensation of workers.

We need to look for a weakness.

(A) Although the economic pie is no longer growing, the portion of the pie allocated to American workers remains unjustly small.

It can certainly be the labour leaders' argument. Even if the pie is not growing, the problem is the unjust distribution of current pie. The redistribution of pie is required even if the pie is not expanding.

(B) If management fails to accommodate the demands of workers, labor leaders will be forced to call strikes that will cripple the operation of industry.

This is not correct. It doesn't focus on logical weakness of the given argument. It threatens dire consequences in case demands are not met.

(C) Although productivity is stagnant, the U.S. population is growing, so that the absolute size of the economic pie continues to grow as well.
The argument states that the economic pie is not growing and that has to be taken to be true since it is a premise. Hence, this is not a logical weakness in the argument. Calling another person a liar is not the same as revealing logical weakness in his/her argument.

(D) As a labor leader, I can be concerned only with the needs of working people, not with the problems faced by management.
Again, this is not a logical weakness in the management's argument. This response is same as saying your problem is not my problem.

(E) The stagnation of U.S. industry has been caused largely by factors—such as foreign competition—beyond the control of American workers.
This reveals the reason for the problems being faced by the economy. It doesn't say what the logical flaw of the argument is.

Answer (A)


Is not "dire consequences" a justified reason for increasing wage?
Manager
Manager
Joined: 23 May 2023
Status:Admissions consultant
Affiliations: MBA Center
Posts: 118
Own Kudos [?]: 50 [0]
Given Kudos: 2
Location: France
Concentration: Entrepreneurship, General Management
GMAT 1: 800 Q51 V51
GPA: 3
WE:Operations (Education)
Send PM
Re: Contrary to the statements of labor leaders, the central economic prob [#permalink]
The statement focusing on the logical weakness in the argument above is:

(A) Although the economic pie is no longer growing, the portion of the pie allocated to American workers remains unjustly small.

This answer choice challenges the argument's assumption that the distribution of wealth is not a central economic problem by pointing out that despite the stagnant or declining productivity, the portion of the economic pie allocated to workers is still unfair. It highlights the logical weakness of assuming that productivity is the sole factor determining the justification for increases in worker compensation.
GMAT Club Bot
Re: Contrary to the statements of labor leaders, the central economic prob [#permalink]
Moderators:
GMAT Club Verbal Expert
6917 posts
GMAT Club Verbal Expert
238 posts
CR Forum Moderator
832 posts

Powered by phpBB © phpBB Group | Emoji artwork provided by EmojiOne